Difference between revisions of "Mock AIME 1 2007-2008 Problems/Problem 6"

(Problem 6)
(Solution)
 
(One intermediate revision by the same user not shown)
Line 2: Line 2:
 
A <math>\frac 1p</math> -array is a structured, infinite, collection of numbers. For example, a <math>\frac 13</math> -array is constructed as follows:  
 
A <math>\frac 1p</math> -array is a structured, infinite, collection of numbers. For example, a <math>\frac 13</math> -array is constructed as follows:  
 
   
 
   
<math>\begin{align*}
+
<cmath>\begin{align*}
 
1 \qquad \frac 13\,\ \qquad \frac 19\,\ \qquad \frac 1{27} \qquad &\cdots\\
 
1 \qquad \frac 13\,\ \qquad \frac 19\,\ \qquad \frac 1{27} \qquad &\cdots\\
 
\frac 16 \qquad \frac 1{18}\,\ \qquad \frac{1}{54} \qquad &\cdots\\
 
\frac 16 \qquad \frac 1{18}\,\ \qquad \frac{1}{54} \qquad &\cdots\\
Line 8: Line 8:
 
\frac 1{216} \qquad &\cdots\\
 
\frac 1{216} \qquad &\cdots\\
 
&\ddots
 
&\ddots
\end{align*}</math>
+
\end{align*}</cmath>
 
   
 
   
 
In general, the first entry of each row is <math>\frac{1}{2p}</math> times the first entry of the previous row. Then, each succeeding term in a row is <math>\frac 1p</math> times the previous term in the same row. If the sum of all the terms in a <math>\frac{1}{2008}</math> -array can be written in the form <math>\frac mn</math>, where <math>m</math> and <math>n</math> are relatively prime positive integers, find the remainder when <math>m+n</math> is divided by <math>2008</math>.
 
In general, the first entry of each row is <math>\frac{1}{2p}</math> times the first entry of the previous row. Then, each succeeding term in a row is <math>\frac 1p</math> times the previous term in the same row. If the sum of all the terms in a <math>\frac{1}{2008}</math> -array can be written in the form <math>\frac mn</math>, where <math>m</math> and <math>n</math> are relatively prime positive integers, find the remainder when <math>m+n</math> is divided by <math>2008</math>.
Line 14: Line 14:
 
== Solution ==
 
== Solution ==
 
Note that the value in the <math>r</math>th row and the <math>c</math>th column is given by <math>\left(\frac{1}{(2p)^r}\right)\left(\frac{1}{p^c}\right)</math>. We wish to evaluate the summation over all <math>r,c</math>, and so the summation will be, using the formula for an infinite [[geometric series]]:
 
Note that the value in the <math>r</math>th row and the <math>c</math>th column is given by <math>\left(\frac{1}{(2p)^r}\right)\left(\frac{1}{p^c}\right)</math>. We wish to evaluate the summation over all <math>r,c</math>, and so the summation will be, using the formula for an infinite [[geometric series]]:
<center><math>\begin{align*}\sum_{r=1}^{\infty}\sum_{c=1}^{\infty} \left(\frac{1}{(2p)^r}\right)\left(\frac{1}{p^c}\right) &= \left(\sum_{r=1}^{\infty} \frac{1}{(2p)^r}\right)\left(\sum_{c=1}^{\infty} \frac{1}{p^c}\right)\\
+
<cmath>\begin{align*}\sum_{r=1}^{\infty}\sum_{c=1}^{\infty} \left(\frac{1}{(2p)^r}\right)\left(\frac{1}{p^c}\right) &= \left(\sum_{r=1}^{\infty} \frac{1}{(2p)^r}\right)\left(\sum_{c=1}^{\infty} \frac{1}{p^c}\right)\\
 
&= \left(\frac{1}{1-\frac{1}{2p}}\right)\left(\frac{1}{1-\frac{1}{p}}\right)\\
 
&= \left(\frac{1}{1-\frac{1}{2p}}\right)\left(\frac{1}{1-\frac{1}{p}}\right)\\
&= \frac{2p^2}{(2p-1)(p-1)}\end{align*}</math></center>
+
&= \frac{2p^2}{(2p-1)(p-1)}\end{align*}</cmath>
 
Taking the denominator with <math>p=2008</math> (indeed, the answer is independent of the value of <math>p</math>), we have <math>m+n \equiv 2008^2 + (2008-1)(2\cdot 2008 - 1) \equiv (-1)(-1) \equiv 1 \pmod{2008}</math> (or consider [[FOIL]]ing). The answer is <math>\boxed{001}</math>.
 
Taking the denominator with <math>p=2008</math> (indeed, the answer is independent of the value of <math>p</math>), we have <math>m+n \equiv 2008^2 + (2008-1)(2\cdot 2008 - 1) \equiv (-1)(-1) \equiv 1 \pmod{2008}</math> (or consider [[FOIL]]ing). The answer is <math>\boxed{001}</math>.
  

Latest revision as of 19:15, 29 January 2020

Problem 6

A $\frac 1p$ -array is a structured, infinite, collection of numbers. For example, a $\frac 13$ -array is constructed as follows:

\begin{align*} 1 \qquad \frac 13\,\ \qquad \frac 19\,\ \qquad \frac 1{27} \qquad &\cdots\\ \frac 16 \qquad \frac 1{18}\,\ \qquad \frac{1}{54} \qquad &\cdots\\ \frac 1{36} \qquad \frac 1{108} \qquad &\cdots\\ \frac 1{216} \qquad &\cdots\\ &\ddots \end{align*}

In general, the first entry of each row is $\frac{1}{2p}$ times the first entry of the previous row. Then, each succeeding term in a row is $\frac 1p$ times the previous term in the same row. If the sum of all the terms in a $\frac{1}{2008}$ -array can be written in the form $\frac mn$, where $m$ and $n$ are relatively prime positive integers, find the remainder when $m+n$ is divided by $2008$.

Solution

Note that the value in the $r$th row and the $c$th column is given by $\left(\frac{1}{(2p)^r}\right)\left(\frac{1}{p^c}\right)$. We wish to evaluate the summation over all $r,c$, and so the summation will be, using the formula for an infinite geometric series: \begin{align*}\sum_{r=1}^{\infty}\sum_{c=1}^{\infty} \left(\frac{1}{(2p)^r}\right)\left(\frac{1}{p^c}\right) &= \left(\sum_{r=1}^{\infty} \frac{1}{(2p)^r}\right)\left(\sum_{c=1}^{\infty} \frac{1}{p^c}\right)\\ &= \left(\frac{1}{1-\frac{1}{2p}}\right)\left(\frac{1}{1-\frac{1}{p}}\right)\\ &= \frac{2p^2}{(2p-1)(p-1)}\end{align*} Taking the denominator with $p=2008$ (indeed, the answer is independent of the value of $p$), we have $m+n \equiv 2008^2 + (2008-1)(2\cdot 2008 - 1) \equiv (-1)(-1) \equiv 1 \pmod{2008}$ (or consider FOILing). The answer is $\boxed{001}$.


With less notation, the above solution is equivalent to considering the product of the geometric series $\left(1+\frac{1}{2 \cdot 2008} + \frac{1}{4 \cdot 2008^2} \cdots\right)\left(1 + \frac{1}{2008} + \frac{1}{2008^2} \cdots \right)$. Note that when we expand this product, the terms cover all of the elements of the array.

By the geometric series formula, the first series evaluates to be $\frac{1}{1 - \frac{1}{2 \cdot 2008}} = \frac{2 \cdot 2008}{2 \cdot 2008 - 1}$. The second series evaluates to be $\frac{1}{1 - \frac{1}{2008}} = \frac{2008}{2008 - 1}$. Their product is $\frac{2008 \cdot 4016}{(2008-1)(2\cdot 2008 - 1)}$, from which we find that $m+n$ leaves a residue of $1$ upon division by $2008$.

See also

Mock AIME 1 2007-2008 (Problems, Source)
Preceded by
Problem 5
Followed by
Problem 7
1 2 3 4 5 6 7 8 9 10 11 12 13 14 15